- Thu Jun 09, 2016 6:49 pm
#26380
Complete Question Explanation
(The complete setup for this game can be found here: lsat/viewtopic.php?t=7717)
The correct answer choice is (C)
The question stem establishes an LG block, and such a block can only occur under Template #2. The appearance of the block creates the following sequential relationship:
J must also appear later than P. Given the positioning of the actors, it is apparent that L must be one of the first three actors to appear (L-G-P, P-L-G, or P-M-L), and thus answer choice (C) is correct.
(The complete setup for this game can be found here: lsat/viewtopic.php?t=7717)
The correct answer choice is (C)
The question stem establishes an LG block, and such a block can only occur under Template #2. The appearance of the block creates the following sequential relationship:
J must also appear later than P. Given the positioning of the actors, it is apparent that L must be one of the first three actors to appear (L-G-P, P-L-G, or P-M-L), and thus answer choice (C) is correct.
You do not have the required permissions to view the files attached to this post.